OBGYN Flashcards

1
Q

A 23-year-old female complains of lower abdominal and pelvic pain, increased vaginal discharge, and postcoital bleeding. Her pain worsens during intercourse, and is accompanied by occasional nausea and vomiting and a feverish feeling. She is sexually active with several male partners.

A physical examination is remarkable for an oral temperature of 38.6°C (101.5°F), cervical motion tenderness, adnexal tenderness without a mass, and a prominent cervical discharge. Office laboratory results include an elevated erythrocyte sedimentation rate and an elevated WBC count. Saline microscopy of vaginal secretions shows abundant numbers of WBCs but is negative for Trichomonas vaginalis and bacterial vaginosis. You order nucleic-acid amplification tests for Chlamydia trachomatis and Neisseria gonorrhoeae.

Which one of the following would be most appropriate at this point? (check one)
- Treatment based on clinical findings
- Treatment when results of testing for Chlamydia trachomatis and Neisseria gonorrhoeae are available
- Transvaginal ultrasonography
- Pelvic CT
- Laparoscopy

A

Treatment based on clinical findings

The diagnosis of pelvic inflammatory disease (PID) is based primarily on the clinical evaluation. Significant consequences can occur if treatment is delayed. Physicians should therefore treat on the basis of clinical judgment without waiting for confirmation from laboratory or imaging tests (SOR B). No single symptom, physical finding, or laboratory test is sensitive or specific enough to definitively diagnose PID (SOR C). Clinical diagnosis alone based on the history, physical examination, and office laboratory results is 87% sensitive, transvaginal ultrasonography is 30% sensitive, and laparoscopy is 81% sensitive but unnecessarily invasive and not cost-effective. A study examining the diagnostic performance of CT in acute PID concluded that the overall sensitivity of CT is poor.

How well did you know this?
1
Not at all
2
3
4
5
Perfectly
2
Q

A 75-year-old female sees you because of a bulge at the vaginal opening. A pelvic examination confirms descent of the vaginal wall to just beyond the hymen. This protrusion is bothering her and interfering with her quality of life. She has had two vaginal deliveries. She is sexually active and has not had any pelvic surgery.

Which one of the following would be the most appropriate initial treatment for this problem? (check one)
- Kegel exercises
- A ring pessary
- A space-occupying pessary
- Hysteropexy
- Hysterectomy

A

A Ring Pessary

Pessaries are considered first-line treatment for pelvic organ prolapse (SOR C). Ring pessaries provide support and are the initial choice in most circumstances. Sexual intercourse can still occur with a ring pessary, which can be inserted and removed by the patient. Space-occupying pessaries are associated with more vaginal discharge and irritation and do not allow for sexual intercourse. While they can improve stress and urge urinary incontinence, Kegel exercises do not treat pelvic organ prolapse. Surgery, including hysterectomy or hysteropexy that conserves the uterus, can be considered after first-line treatment with a pessary.

How well did you know this?
1
Not at all
2
3
4
5
Perfectly
3
Q

A 34-year-old female consults you because of excessive body and facial hair. She has a normal body weight, no other signs of virilization, and regular menses. She had a bilateral tubal ligation 4 years ago.

Which one of the following would be the most appropriate treatment for her mild hirsutism? (check one)
Leuprolide
Metformin (Glucophage)
Prednisone
Spironolactone (Aldactone)

A

Spironolactone (Aldactone)

Antiandrogens such as spironolactone, along with oral contraceptives, are recommended for the treatment of hirsutism in premenopausal women (SOR C). Women should avoid becoming pregnant while on spironolactone because of the potential for teratogenic effects. In addition to having side effects, prednisone is only minimally helpful for reducing hirsutism by suppressing adrenal androgens. Leuprolide, although better than placebo, has many side effects and is expensive. Metformin can be used to treat patients with polycystic ovary syndrome, but this patient does not meet the criteria for this diagnosis.

How well did you know this?
1
Not at all
2
3
4
5
Perfectly
4
Q

A 17-year-old female with a history of morbid obesity sees you to discuss contraceptive options. She is heterosexual and is currently sexually active with one male partner. She has heavy irregular periods and associated anemia and is interested in a contraceptive option that will both provide reliable birth control and decrease her menstrual blood loss. She recently had negative tests for HIV, gonorrhea, and Chlamydia at a local health department. Her examination is unremarkable except for a weight of 136 kg (300 lb) and a BMI of 50 kg/m2.

Which one of the following would be the best option for contraception for this patient? (check one)
A diaphragm with spermicide
The norelgestromin/ethinyl estradiol transdermal system (Ortho Evra)
The levonorgestrel-releasing intrauterine system (Mirena IUD)
Medroxyprogesterone acetate (Depo-Provera)

A

The levonorgestrel-releasing intrauterine system (Mirena IUD)

This patient has heavy menstrual bleeding, associated anemia, and morbid obesity, all of which need to be taken into consideration when choosing contraception. Medroxyprogesterone acetate can contribute to weight gain and thus should not be the first choice in this individual. The norelgestromin/ethinyl estradiol transdermal system is not recommended in patients with a weight over 90 kg and thus is not an option for this patient. The levonorgestrel-releasing intrauterine system would be the best option, given the associated significant decrease in menstrual blood loss after the first 3 months of insertion and equal effectiveness in obese and non-obese patients. Although a diaphragm is an option, it will not decrease her menstrual blood loss.

How well did you know this?
1
Not at all
2
3
4
5
Perfectly
5
Q

A 30-year-old otherwise healthy female has concerns about her menses and fertility. Her last menstrual period was 8 months ago when she stopped taking oral contraceptive pills (OCPs). In her teens and early twenties she had irregular, sporadic periods. Four years ago she developed menometrorrhagia and resultant iron deficiency anemia; this was corrected with the use of OCPs. She is now interested in becoming pregnant. Her physical examination, including a gynecologic examination, is normal. A urine pregnancy test is negative and her TSH level is in the normal range.

Which one of the following is the most appropriate next step? (check one)
A CBC and metabolic panel
Serum LH and FSH levels
Karyotype analysis
Pelvic ultrasonography

A

Serum LH and FSH levels

This patient suffers from secondary amenorrhea (defined as the cessation of regular menses for 3 months or irregular menses for 6 months). The most common causes of secondary amenorrhea are polycystic ovary syndrome, primary ovarian failure, hypothalamic amenorrhea, and hyperprolactinemia. With a normal physical examination, negative pregnancy test, and no history of chronic disease, a hormonal workup is indicated, including TSH, LH, and FSH levels (SOR C).

A hormonal challenge with medroxyprogesterone to provoke withdrawal bleeding is used to assess functional anatomy and estrogen levels (SOR C). However, it has poor specificity and sensitivity for ovarian function and a poor correlation with estrogen levels.

Pelvic ultrasonography is indicated in the workup of primary amenorrhea to confirm the presence of a uterus and detect structural abnormalities of the reproductive organs. Likewise, karyotyping can be used for patients with primary amenorrhea, as conditions such as Turner’s syndrome and androgen insensitivity syndrome are due to chromosomal abnormalities.

A CBC and metabolic panel would not be initial considerations in the workup of amenorrhea unless the patient has a known chronic disease which may affect the results.

How well did you know this?
1
Not at all
2
3
4
5
Perfectly
6
Q

A 24-year-old gravida 4 para 2 with mild chronic hypertension and an uncomplicated pregnancy has just delivered a vigorous male by spontaneous vaginal delivery. She is noted to have heavy vaginal bleeding and a bimanual examination reveals a soft, poorly contracted uterus. Her temperature is 37.1°C (98.8°F), blood pressure 158/92 mm Hg, pulse rate 105 beats/min, and oxygen saturation 95% on room air.

Which one of the following uterotonic agents is CONTRAINDICATED in the management of this patient’s postpartum hemorrhage? (check one)
Oxytocin (Pitocin)
Methylergonovine
Carboprost tromethamine (Hemabate)
Misoprostol (Cytotec

A

Methylergonovine

Uterotonics are the first-line treatment for postpartum hemorrhage in patients with decreased uterine tone. While all of the uterotonic agents listed are options for the management of postpartum hemorrhage, methylergonovine should be avoided if the patient is hypertensive. It is an ergot alkaloid that causes generalized smooth muscle contraction and can raise blood pressure. Oxytocin and misoprostol do not have any contraindications. Carboprost tromethamine should be avoided in asthmatic patients and is relatively contraindicated if the patient has hepatic, renal, or cardiac disease.

How well did you know this?
1
Not at all
2
3
4
5
Perfectly
7
Q

A 23-year-old female presents with menstrual irregularity, increased facial hair, and acne. Your evaluation leads to a diagnosis of polycystic ovary syndrome.

Which one of the following is the first-line management for her constellation of symptoms? (check one)
Clomiphene (Clomid)
Hormonal contraceptives
Metformin (Glucophage)
Pioglitazone (Actos)
Spironolactone (Aldactone)

A

Hormonal contraceptives

Hormonal contraceptives are the first-line therapy for menstrual abnormalities, hirsutism, and acne in polycystic ovary syndrome. Clomiphene is used for infertility. Thiazolidinediones have an unfavorable risk-benefit ratio overall. Metformin is beneficial for metabolic/glycemic abnormalities and menstrual irregularities, but does not improve hirsutism or acne. Spironolactone may be used as an add-on to hormonal contraceptives for treatment of hirsutism and acne.

How well did you know this?
1
Not at all
2
3
4
5
Perfectly
8
Q

In a woman whose group B Streptococcus status is unknown, which one of the following is a risk factor requiring empiric intrapartum antibiotic prophylaxis against early-onset group B streptococcal infection in her newborn? (check one)
Fetal tachycardia
Delivery at less than 35 weeks gestation
Rupture of the membranes 12 hours before delivery
Gestational diabetes during the pregnancy
Use of vacuum extraction during delivery

A

Delivery at less than 35 weeks gestation

Of the choices listed, prematurity is the greatest risk factor for group B streptococcal infection. The most important risk would be signs or symptoms of sepsis in a neonate. The other conditions listed are not risk factors for early-onset GBS in neonates.

How well did you know this?
1
Not at all
2
3
4
5
Perfectly
9
Q

A 25-year-old female reports the absence of menses for the past 6 months. She is currently not taking any medications. You confirm that she is not pregnant and order additional laboratory testing. TSH, LH, and FSH levels are normal but she has an elevated prolactin level.

Which one of the following would be most appropriate at this point to further evaluate her pituitary gland? (check one)
A follow-up serum prolactin level in 4 weeks
A prolactin-stimulating hormone level
MRI of the pituitary
Head CT with intravenous contrast

A

MRI of the pituitary

Prolactin levels can be elevated because of a pituitary adenoma, medication side effects, hypothyroidism, or a mass lesion compromising normal hypothalamic inhibition. Elevated prolactin levels inhibit the secretion and effect of gonadotropins. In almost all patients with an elevated prolactin level, MRI of the pituitary is recommended to exclude the possibility of a pituitary adenoma (SOR C). This patient is not on any medications, essentially ruling out a pharmacologic trigger for her elevated prolactin.

How well did you know this?
1
Not at all
2
3
4
5
Perfectly
10
Q

A 24-year-old gravida 2 para 1 presents to your office for her first prenatal visit at 7 weeks gestation. You review her vaccine records and note that she received Tdap 1 year ago.

When should you recommend that she get her next Tdap? (check one)
Post partum
At this visit
Anytime after the first trimester
Between 27 and 36 weeks gestation
10 years after the last dose

A

Between 27 and 36 weeks gestation

Due to the increasing incidence of pertussis, the Centers for Disease Control and Prevention recommends that all pregnant women receive Tdap vaccine during every pregnancy regardless of when their last dose was. It is ideally administered between 27 and 36 weeks gestation to maximize the maternal antibody response and passive antibody transfer to the infant.

How well did you know this?
1
Not at all
2
3
4
5
Perfectly
11
Q

A 25-year-old primigravida presents at 28 weeks gestation for a routine prenatal visit. She is undecided about breastfeeding versus bottle feeding and asks if breastfeeding provides any benefits for her own health.

You advise her that breastfeeding would decrease her risk of later developing (check one)
colon cancer
diabetes mellitus
lung cancer
osteoarthiritis
recurrent respiratory infections

A

diabetes mellitus

Breastfeeding provides many health benefits to both the mother and the infant. Maternal benefits include a decreased risk of developing cardiometabolic disease, including diabetes mellitus, hypertension, and cardiovascular disease; a decreased risk of breast cancer and ovarian cancer; and a decreased risk of postpartum depression. A link has not been established between breastfeeding and a reduced risk of developing colon cancer or lung cancer or osteoarthritis later in life. While breastfeeding may reduce the infant’s risk of respiratory disease, this is not an expected benefit for the mother.

How well did you know this?
1
Not at all
2
3
4
5
Perfectly
12
Q

A 32-year-old primigravida at 36 weeks gestation complains of headaches. She denies vaginal bleeding, leakage of fluid, and contractions, and the fetus is moving normally. Her blood pressure is 155/100 mm Hg and a urinalysis shows 4+ protein. The rest of her examination is normal and a cervical examination shows 1 cm of dilation, 50% effacement, a soft consistency, anterior position, and –2 vertex station. Results of a preeclampsia panel are all in the normal range.

Which one of the following is the most appropriate management for this patient? (check one)
Start labetalol (Trandate) and discharge home on bed rest with close follow-up
Start magnesium sulfate and induce labor now
Start magnesium sulfate, administer corticosteroids, and induce labor in 48 hours
Start magnesium sulfate, lower blood pressure to 140/90 mm Hg, and induce labor at 37 weeks gestation
Arrange for urgent cesarean section

A

Start magnesium sulfate and induce labor now

This patient likely has severe preeclampsia based on her elevated blood pressure with 4+ protein on her urinalysis. Patients with severe preeclampsia near term should be placed on magnesium sulfate to prevent seizures, and labor should be induced immediately. An urgent cesarean section is not necessary. Corticosteroids have not been shown to improve neonatal outcomes when given after 34 weeks gestation. Elevated blood pressures can be managed with hydralazine and labetalol. Normalizing blood pressure is not recommended, but these drugs should be used when blood pressure is over 160/105 mm Hg.

How well did you know this?
1
Not at all
2
3
4
5
Perfectly
13
Q

A 35-year-old female presents for contraceptive counseling. Her last menstrual period was 3 weeks ago and she had unprotected sex 2 days ago. A pregnancy test is negative.

Which one of the following would be the most effective emergency contraceptive agent for this patient? (check one)
Oral levonorgestrel (Plan B One-Step)
Oral ulipristal (Ella)
Subcutaneous depot medroxyprogesterone acetate (Depo-Provera)
An etonogestrel subdermal implant (Nexplanon)
A copper IUD (Paragard)

A

A copper IUD (Paragard)

The copper IUD is the most effective form of emergency contraception with a pregnancy rate of 0.1%, followed by oral ulipristal with a pregnancy rate of 1.3%. Oral levonorgestrel is less effective than both with a pregnancy rate of 2.5%. Subcutaneous depot medroxyprogesterone acetate and the etonogestrel subdermal implant are not recommended as emergency contraceptives.

How well did you know this?
1
Not at all
2
3
4
5
Perfectly
14
Q

A 25-year-old female with hypothyroidism sees you for preconception counseling. Her thyroid problem has been well managed with levothyroxine (Synthroid), 75 !g daily, but she asks your advice about changing her treatment to something more natural now that she is planning to become pregnant.

Which one of the following is the best recommendation for this patient? (check one)
Continue the current dosage of levothyroxine
Reduce the current dosage of levothyroxine to 50 !g daily
Change to a comparable dosage of combination levothyroxine/L-triiodothyronine
Change to a comparable dosage of desiccated thyroid

A

Continue the current dosage of levothyroxine

Untreated hypothyroidism during pregnancy impairs fetal development and increases the risk of spontaneous miscarriage, prematurity, preeclampsia, gestational hypertension, and postpartum hemorrhage. These risks are mitigated by appropriate levothyroxine treatment. Levothyroxine/L¬triiodothyroxine combinations and desiccated thyroid preparations have the potential to correct maternal hypothyroidism, but the T4 level may still be too low to provide the transplacental delivery necessary for optimal fetal health. The most appropriate pregnancy planning advice is to continue the current dosage of levothyroxine with a plan for monthly monitoring of TSH and T4 during pregnancy, with the expectation that an increase in dosage may be required as the pregnancy progresses.

How well did you know this?
1
Not at all
2
3
4
5
Perfectly
15
Q

Which one of the following is consistent with best practices for prescribing hormonal contraception? (check one)
Delaying initiation until the patient’s next menstrual period to avoid incidental use in early pregnancy
Limiting prescription refills to 3 months at a time
Obtaining a thorough medical history to screen for contraindications
Requiring in-person visits rather than telehealth visits for contraceptive counseling due to the need for a gynecologic examination
Requiring patients to have an up-to-date Papanicolaou test and sexually transmitted infection screening

A

Obtaining a thorough medical history to screen for contraindications

There are many patient-related historical factors that may affect the safety and choice of hormonal contraception, but very few physical factors are likely to be found on examination that would not otherwise have been identified. Obtaining a thorough medical history is standard practice, but the Choosing Wisely campaign recommends against requiring a pelvic or other physical examination prior to prescribing oral contraceptives. It is unnecessary to wait to begin hormonal contraception until after the next menses, as inadvertent exposure to oral contraception will not harm an early pregnancy. Prescribing a 1-year supply of hormonal contraceptives improves adherence and lowers cost. There is broad consensus that sexually transmitted infection screening and Papanicolaou testing should not be required to prescribe contraception.

How well did you know this?
1
Not at all
2
3
4
5
Perfectly
16
Q

A 50-year-old male with difficult-to-control hypertension seeks your advice regarding progressive breast enlargement. Your examination reveals bilateral firm, glandular tissue in a concentric mass around the nipple-areola complex. You diagnose gynecomastia.

Which one of the following antihypertensive medications is most likely to cause this problem? (check one)
Doxazosin (Cardura)
Hydrochlorothiazide
Lisinopril (Prinivil, Zestril)
Losartan (Cozaar)
Spironolactone (Aldactone)

A

Spironolactone (Aldactone)

Except for persistent pubertal gynecomastia, medication use and substance use are the most common causes of nonphysiologic gynecomastia. Common medication-related causes include the use of antipsychotic agents, antiretroviral drugs, or prostate cancer therapies. Spironolactone also has a high propensity to cause gynecomastia; other mineralocorticoid receptor antagonists, such as eplerenone, have not been associated with similar effects. Discontinuing the contributing agent often results in regression of breast tissue within 3 months.

How well did you know this?
1
Not at all
2
3
4
5
Perfectly
17
Q

A 58-year-old postmenopausal female presents with a recent onset of painless vaginal bleeding. Her last menses occurred 8 years ago and she has had no bleeding until now. She reports that her Papanicolaou smears have always been normal, with the last one obtained a year ago. A pelvic examination today is normal.

Which one of the following management options is the preferred next diagnostic step? (check one)
Colposcopy with endocervical curettage
Transvaginal ultrasonography
Saline infusion sonohysterography
Hysteroscopy

A

Transvaginal ultrasonography

Transvaginal ultrasonography is the preferred initial test for a patient with painless postmenopausal bleeding, although endometrial biopsy is an option if transvaginal ultrasonography is not available. Transvaginal ultrasonography showing an endometrial thickness <3–4 mm would essentially rule out endometrial carcinoma (SOR C). An endometrial biopsy is invasive and has low sensitivity for focal lesions. Saline infusion hysterography should be considered if the endometrial thickness is greater than the threshold, or if an adequate measurement cannot be obtained by ultrasonography. If hysterography shows a global process, then a histologic diagnosis can usually be obtained with an endometrial biopsy, but if a focal lesion is present hysteroscopy should be considered as the next diagnostic step. Colposcopy is not indicated given the patient’s normal Papanicolaou smear.

How well did you know this?
1
Not at all
2
3
4
5
Perfectly
18
Q

A 39-year-old female presents with lower abdominal/pelvic pain. On examination, with the patient in a supine position, you palpate the tender area of her lower abdomen. When you have her raise both legs off the table while you palpate the abdomen, her pain intensifies.

Which one of the following is the most likely diagnosis? (check one)
Appendicitis
A hematoma within the abdominal wall musculature
Diverticulitis
Pelvic inflammatory disease
An ovarian cyst

A

A hematoma within the abdominal wall musculature

A reduction of the pain caused by abdominal palpation when the abdominal muscles are tightened is known as Carnett’s sign. If the cause of the pain is visceral, the taut abdominal muscles may protect the locus of pain. In contrast, intensification of pain with this maneuver points to a source of pain within the abdominal wall itself.

How well did you know this?
1
Not at all
2
3
4
5
Perfectly
19
Q

A 50-year-old female reports vaginal dryness, burning, and pain with penetration during sexual intercourse. On examination she is noted to have pale, dry vaginal epithelium that is smooth and shiny with loss of most rugation.

Which one of the following treatments is most likely to be effective for her sexual dysfunction? (check one)
Cognitive-behavioral therapy
Vaginal estrogen
Testosterone therapy
Bupropion (Wellbutrin)
Sildenafil (Viagra)

A

Vaginal estrogen

This patient has genitourinary syndrome of menopause (formerly termed vulvovaginal atrophy) based on her symptoms and examination. Estrogen therapy is highly effective for dyspareunia related to genitourinary syndrome of menopause, with the vaginal route preferred over systemic therapy if vaginal dryness is the primary concern. Bupropion and sildenafil may benefit women with sexual dysfunction induced by antidepressant medications. Data on the benefit of testosterone therapy is limited and inconsistent and lacks long-term information about safety. Cognitive-behavioral therapy has been shown to effectively treat low sexual desire, but does not affect the physiologic changes associated with genitourinary syndrome of menopause.

How well did you know this?
1
Not at all
2
3
4
5
Perfectly
20
Q

Which one of the following is the basis for the most effective method of natural family planning? (check one)
Calendar calculation
Basal body temperature charting
Cervical mucus monitoring
Monitoring for urine estrogen metabolites
Coitus interruptus (withdrawal)

A

Cervical mucus monitoring

Natural family planning (NFP) is a potentially effective method for contraception and for determining the time of ovulation for purposes of conception. While the contraceptive effectiveness of the different NFP methods varies significantly, the success rates for typical use are as high as 92%–98% (SOR B). Monitoring the presence and consistency of cervical mucus production allows for the determination of both the beginning and end of a woman’s most fertile period. Some NFP methods use cervical mucus secretion as the sole basis for determining fertility. The symptothermal method also incorporates calendar calculations, basal body temperature measurement, and ovulation-related symptoms as a complement to the cervical mucus component. The Marquette Model incorporates cervical mucus and basal body temperature charting with electronic monitoring of urine estrogen and LH metabolites to provide additional information to determine when ovulation has occurred.

How well did you know this?
1
Not at all
2
3
4
5
Perfectly
21
Q

A 32-year-old white primigravida has a stillbirth at 33 weeks gestation. Which one of the following is the most likely cause? (check one)
Infection
Placental disease
A fetal structural disorder
A hypertensive disorder

A

Placental disease

Stillbirth is defined as fetal death occurring at or after 20 weeks gestation, and affects approximately 1 in 160 pregnancies in the United States. A large study of stillbirths from 2006 to 2008 tried to establish a cause in 663 cases, and a probable or possible cause was identified in approximately 75% of these. While there were some significant ethnic differences, placental abnormalities and obstetric complications were the largest category of causes in white women, and this was even more true after 32 weeks gestation. Other important causes included infection and fetal defects. More than one cause was found in one-third of cases.

How well did you know this?
1
Not at all
2
3
4
5
Perfectly
22
Q

A 24-year-od nulligravida comes to your office for contraception counseling. She has a seizure disorder that is well controlled on carbamazepine (Tegretol). She is a nonsmoker and has no other medical problems or complaints. She is currently in a relationship and does not want to get pregnant in the next several years.

Which one of the following contraceptive options would be the most appropriate? (check one)
Progestin-only pills
Combined oral contraceptives
The etonogestrel/ethinyl estradiol vaginal ring (NuvaRing
The norelgestromin/ethinyl estradiol contraceptive patch (Ortho Evra)
A levonorgestrel intrauterine device (Mirena)

A

A levonorgestrel intrauterine device (Mirena)

Certain antiepileptic drugs induce hepatic metabolism of estrogen and progestin (carbamazepine, oxcarbazepine, phenobarbital, phenytoin, and topiramate). This can potentially lead to failure of any contraceptive that contains estrogen and progestin. Progestin-only pills are most effective in women who are exclusively breastfeeding. They are not as effective in pregnancy prevention in other circumstances. Another effective option for women taking antiepileptic medications would be an intrauterine device. The levonorgestrel (progestin only) IUD and copper IUD are acceptable choices even for a nulligravida. The single-rod implantable progestin system also would be an acceptable choice for this patient.

How well did you know this?
1
Not at all
2
3
4
5
Perfectly
23
Q

A 46-year-old African-American female sees you because of a history of excessive uterine bleeding and irregularity in her menstrual cycle. She has three children and had a tubal ligation after her last delivery. A pelvic examination does not reveal any pathology to explain her symptoms. Further laboratory evaluation indicates that she is mildly anemic. You perform an endometrial biopsy in the office that confirms your suspicion of endometrial hyperplasia without atypia.

Which one of the following is the treatment of choice for this patient?
(check one)
Elective hysterectomy
Hysteroscopic endometrial laser ablation
High-dose oral estrogen supplementation
Antifibrinolytic therapy
Progestational drugs

A

Progestational drugs

Medical therapy with progestational drugs is the treatment of choice for menorrhagia due to endometrial hyperplasia without atypia. Progestins convert the proliferative endometrium to a secretory one, causing withdrawal bleeding and the regression of hyperplasia. The most commonly used form is cyclic oral medroxyprogesterone, given 14 days per month, but implanted intrauterine levonorgestrel is the most effective (SOR A) and also provides contraception.
High-dose estrogen supplementation would further stimulate the endometrium. Estrogen is useful in cases where minimal estrogen stimulation is associated with breakthrough bleeding. The anti-fibrinolytic agent tranexamic acid prevents the activation of plasminogen and is given at the beginning of the cycle to decrease bleeding. Side effects and cost limit this treatment option, however. It may be most useful in women with bleeding disorders or with contraindications to hormonal therapy.
NSAIDs, which decrease prostaglandin levels, reduce menstrual bleeding but not as effectively as progestins. While mefenamic acid is marketed for menstrual cramps and bleeding, all NSAIDs have a similar effect in this regard.
If medical management fails, hysteroscopic endometrial ablation is an option for reducing uterine bleeding but is considered permanent and obviously will impair fertility. Hysterectomy is reserved for severe and chronic bleeding that is not relieved by other measures.

How well did you know this?
1
Not at all
2
3
4
5
Perfectly
24
Q

A 27-year-old gravida 2 para 2 presents because of tenderness in her left lower breast, which she first noticed this morning. Three weeks ago she vaginally delivered an 8 lb 1 oz infant. She breastfed her first child for 10 months and initiated breastfeeding after this delivery without difficulty. Currently she is feeding her infant on cue about every 2–4 hours. On examination she has 4 cm of focal tenderness at 6 o’clock on the breast with no skin erythema. Her vital signs include a temperature of 37.0°C (98.6°F), a pulse rate of 84 beats/min, and a blood pressure of 118/72 mm Hg.

Which one of the following would be the most appropriate next step? (check one)
A trial of conservative management of breastfeeding on cue and analgesics
Expressing breast milk by hand every 1½ hours to keep the breast emptied
Expressing breast milk with a breast pump hourly to keep the breast emptied
Amoxicillin/clavulanate (Augmentin), 875/125 mg twice daily for 5 days; plus pumping and discarding the breast milk
Cephalexin, 500 mg four times daily for 7 days; plus pumping and discarding the breast milk

A

A trial of conservative management of breastfeeding on cue and analgesics

This patient has mild lactational mastitis with no systemic symptoms. Antibiotics are unnecessary and the condition may be managed conservatively with rest, cold compresses, over-the-counter acetaminophen or NSAIDs, and close monitoring for systemic symptoms. The patient should continue on-demand breastfeeding (physiologic feeding). Efforts to keep the breast drained of milk, such as frequent breast pumping or decreased feeding intervals, increase milk production, which may increase breast pain or abscess formation. If the patient develops fever or chills, antibiotics covering Staphylococcus and Streptococcus should be started. The standard treatment is dicloxacillin, 500 mg four times daily for 10–14 days. Other commonly used antibiotics are amoxicillin/clavulanate, 875/125 mg twice daily, or cephalexin, 500 mg four times daily, for 10–14 days. Breast milk does not need to be discarded when the patient is taking these antibiotics.

How well did you know this?
1
Not at all
2
3
4
5
Perfectly
25
Q

A 44-year-old female with localized breast cancer is receiving counseling about adjuvant long-term therapy. Which one of the following is more likely to occur with an aromatase inhibitor such as letrozole (Femara) than with tamoxifen (Soltamox)?

(check one)
Endometrial cancer
Venous thromboembolism
Inflammatory arthritis
Myalgias

A

Myalgias

Myalgias and noninflammatory arthralgias are more likely with aromatase inhibitors. Venous thromboembolism rarely occurs with these drugs. Endometrial cancer may occur with long-term use of tamoxifen.

How well did you know this?
1
Not at all
2
3
4
5
Perfectly
26
Q

Which class of medication is first-line therapy for uncomplicated depression during pregnancy?
(check one)
Monoamine oxidase inhibitors (MAOIs)
SSRIs
SNRIs
Stimulants
Tricyclic antidepressants

A

SSRIs

The treatment of depression in pregnancy is determined by the severity of the symptoms and any past history of treatment response. For women who have a new onset of mild or moderate depression, it may be best to start with nonpharmacologic treatments such as supportive psychotherapy or cognitive-behavioral therapy. These interventions may improve the depression enough that the patient will not need medications. However, in situations where pharmacologic treatment is clearly indicated, SSRIs are thought to have the best safety profile. Fluoxetine, sertraline, and citalopram have extensive data to support their safety in pregnancy and should be considered first line. Paroxetine is the one SSRI that is thought to carry an increased risk of congenital malformations with first-trimester exposure and should be avoided.

Tricyclic antidepressants are class D in pregnancy. SNRIs do not have as much safety data as SSRIs to support their use in pregnancy and would be considered a second-line choice. MAOIs are known teratogens and should be avoided in pregnancy. Stimulants are not first-line agents and should be avoided in pregnancy.

How well did you know this?
1
Not at all
2
3
4
5
Perfectly
27
Q

Which one of the following is NOT a risk factor for stillbirth? (check one)
Smoking
Advanced maternal age
Congential anomalies
Vigorous exercise
BMI >30 kg/m2

A

Vigorous exercise

Risk factors for stillbirth include advanced maternal age, smoking >½ pack of cigarettes a day, congenital
anomalies, and a BMI >30 kg/m2. Excessive exercise has not been shown to increase the risk for
stillbirth.

How well did you know this?
1
Not at all
2
3
4
5
Perfectly
28
Q

A morbidly obese 68-year-old male complains of breast enlargement. He has not noticed any pain or discomfort from this problem. His past medical history is negative except for type 2 diabetes mellitus and hypertension. His medications include metformin (Glucophage), 1000 mg twice daily; lisinopril (Prinivil, Zestril), 20 mg daily; and aspirin, 81 mg daily. His family history is negative for breast cancer. A physical examination is negative except for a BMI of 45 kg/m2 and symmetric bilateral adipose tissue in the breast region on inspection and palpation. There is no glandular tissue on careful palpation of the area beneath the areolae and nipples. No nodules or axillary nodes are detected. There is no nipple retraction or discharge, and no skin changes.

Which one of the following is the most likely cause of this problem?
(check one)
Fat necrosis
Gynecomastia
Pseudogynecomastia
Breast cancer
Mastitis

A

Pseudogynecomastia

This patient most likely has pseudogynecomastia due to increases in subareolar fat secondary to his obesity. This is based upon clinical findings of symmetric adipose tissue in the breast region bilaterally and a lack of firm, palpable glandular tissue in the nipple and areolar region. In gynecomastia, there is palpable, firm glandular tissue in a concentric mass around the nipple-areola complex. Hard, immobile masses, masses associated with skin changes, nipple retraction, nipple discharge, or enlarged lymph nodes would suggest possible malignancy. Fat necrosis would involve a history of breast region trauma and would generally be asymmetric. Mastitis would cause clinical signs of infection.

How well did you know this?
1
Not at all
2
3
4
5
Perfectly
29
Q

You respond to a code blue in the obstetrics department. The patient is a 19-year-old primigravida at 35 weeks gestation, hospitalized with severe preeclampsia. A nurse anesthetist has placed an oral airway and is administering 100% oxygen to the apneic patient. She reports no difficulty ventilating the patient with a bag and valve, and no gagging with oral airway insertion. The patient’s blood pressure is 100/60 mm Hg and her pulse rate is 70 beats/min and regular. Her pupils are equal and sluggishly reactive, and she is flaccid and areflexic. The patient had been treated with a magnesium sulfate infusion and a recent bolus of labetalol.

Which one of the following medications should you administer initially?
(check one)
Calcium gluconate
Fosphenytoin
Labetalol
Lorazepam (Ativan)
Dopamine

A

Calcium gluconate

During the treatment of severe preeclampsia with intravenous magnesium, the occurrence of apnea and areflexia is most consistent with magnesium toxicity. In addition to hemodynamic support, calcium infusion is recommended as an antidote. Calcium chloride can be used if a central line has been established. Calcium gluconate would be safer with a peripheral intravenous site.
Lorazepam, phenytoin, and fosphenytoin are less useful in preventing eclamptic seizures than magnesium. Labetalol is not indicated given the patient’s current blood pressure level. Dopamine, a pressor agent, is not indicated in this scenario, and could aggravate the patient’s preeclampsia.

How well did you know this?
1
Not at all
2
3
4
5
Perfectly
30
Q

A 30-year-old female complains of dysmenorrhea, pelvic pain, and dyspareunia. Which one of
the following would be appropriate to detect endometriosis?
(check one)
A CA-125 assay
Transvaginal ultrasonography
CT of the pelvis
MRI of the pelvis
Colonoscopy

A

Transvaginal ultrasonography

Endometriosis is caused by menstrual tissue in the pelvic peritoneal cavity. Infertility, dysmenorrhea, and
dyspareunia with postcoital bleeding are common. Although laparoscopy with histology is the definitive
test, transvaginal ultrasonography is the noninvasive test of choice. CA-125 will often be elevated but is
nonspecific. CT and MRI also have low specificity, and colonoscopy is of no value in the evaluation of
endometriosis.

How well did you know this?
1
Not at all
2
3
4
5
Perfectly
31
Q

In women with polycystic ovary syndrome, the risk is increased the most for carcinoma of the?

(check one)
Breast
Cervix
Colon
Endometrium
Ovary

A

Endometrium

Several disorders that are common in women with polycystic ovary syndrome are associated with an increased risk for endometrial carcinoma, including obesity, hyperinsulinemia, diabetes mellitus, anovulatory cycles, and high androgen levels.

How well did you know this?
1
Not at all
2
3
4
5
Perfectly
32
Q

A 25-year-old primigravida asks about pain management during labor. You inform her that use of regional analgesia during labor?

(check one)
Increases the likelihood of cesarean delivery
Increases the risk for instrument-assisted vaginal delivery
Provides less pain relief than opioid analgesia
Lowers 1-minute Apgar scores

A

Increases the risk for instrument-assisted vaginal delivery

Regional analgesia in laboring patients increases the risk of vacuum-or forceps-assisted delivery (relative risk = 1.42; 95% confidence interval, 1.28–1.57; 23 trials; n = 735). Multiple randomized, controlled trials have compared regional analgesia with no analgesia. In a meta-analysis, no statistically significant impact was found on the risk of cesarean delivery, maternal satisfaction with pain relief, long-term backache, or immediate effect on neonatal status as determined by Apgar scores. Regional analgesia provides better pain relief than opioid analgesia.

How well did you know this?
1
Not at all
2
3
4
5
Perfectly
33
Q

Which one of the following conditions presents an unacceptable health risk for combined oral contraceptive use?

(check one)
Migraine with aura
Endometrial hyperplasia
Breastfeeding 1–6 months post partum
Chronic hepatitis
Previous laparoscopic banding weight-loss surgery

A

Migraine with aura

The World Health Organization (WHO) publishes the medical eligibility criteria for contraceptive use as a guideline for the appropriate use of contraceptives. There are four categories that define the appropriateness of contraceptive use in women with certain medical problems:

Category 1: A condition for which there is no restriction for the use of the contraceptive method.
Category 2: A condition for which the advantages of using the method generally outweigh the theoretical or proven risks.
Category 3: A condition for which the theoretical risk or proven risks usually outweigh the advantages of using the method.
Category 4: A condition that represents an unacceptable health risk if the contraceptive method is used.

A history of migraine with aura is classified as category 4 for oral contraceptives. Women with a history of migraines are 2–4 times as likely to have a stroke compared to women without migraines, and women who have an aura associated with their migraines are at even higher risk. Migraine without aura is classified as category 2 in women younger than 35 and category 3 in women 35 or older. Nonmigrainous headaches are category 1, as is chronic hepatitis C.

Combined oral contraceptive use does not appear to increase the rate or severity of cirrhotic fibrosis and there is no increased risk for hepatocellular carcinoma. Combined oral contraceptives are not recommended for use in women with acute hepatitis C. Breastfeeding is considered category 2 by the CDC and category 3 by WHO. There is conflicting evidence about the effects on the volume of breast milk in women who are on combined oral contraceptives, but the concerns are mainly during the first month of the postpartum period. There have not been any demonstrated adverse health effects in infants exposed to combined oral contraceptives through breast milk. Laparoscopic banding weight-loss surgery is category 1 for combined oral contraceptive use. Evidence shows no significant decrease in the effectiveness of oral contraceptives in women who have had this surgery. Combined oral contraceptive use in patients with endometrial hyperplasia is category 1. Combined oral contraceptives have been used to decrease the risk for endometrial cancer.

How well did you know this?
1
Not at all
2
3
4
5
Perfectly
34
Q

Which one of the following is a contraindication to the use of combined hormonal contraceptives?

(check one)
A family history of breast cancer in a first degree relative
Rheumatoid arthritis treated with immunosuppression
Morbid obesity
Migraine headaches with aura
Ovarian cancer

A

Migraine headaches with aura

The U.S. Medical Eligibility Criteria for Contraceptive Use were created to guide health care providers in assessing the safety of contraceptive use for patients with specific conditions. Category 1 includes conditions for which no restrictions exist for use of the contraceptive method. Category 2 indicates that the method generally can be used, but careful follow-up may be required. Category 3 is used to classify conditions for which the method usually is not recommended unless more-preferred methods are not available or acceptable. Category 4 comprises conditions that represent an unacceptable health risk if the method is used. For combined hormonal contraceptives, migraine headaches with aura at any age are classified as category 4 because of the increased risk of ischemic stroke.

35
Q

A 26-year-old gravida 2 para 1 at 10 weeks’ gestation presents to the emergency department with abdominal pain and vaginal spotting. Ultrasonography reveals an ectopic pregnancy. Her blood type is A-negative, antibody-negative. Appropriate management with regard to her Rh status includes: (check one)
Administration of 50 µg of RHO immune globulin (RhoGAM)
Administration of 300 µg of RhoGAM
Administration of 50 µg of RhoGAM only if she requires laparoscopic intervention
Administration of 300 µg of RhoGAM only if she requires laparoscopic intervention
No RhoGAM, as it is not indicated in an Rh-negative woman with an ectopic pregnancy

A

Administration of 50 µg of RHO immune globulin (RhoGAM)

Both ectopic pregnancy and spontaneous or therapeutic abortion pose a significant risk for fetomaternal hemorrhage. Thus, administration of RHO immune globulin (RhoGAM) is recommended in any Rh-negative patient who is unsensitized (D antibody screen–negative prior to administration of RhoGAM). If the estimated gestational age is 12 weeks or less, 50 mcg of RhoGAM is recommended. If the estimated gestational age is greater than 12 weeks, 300 µg of RhoGAM is recommended

36
Q

Metformin (Glucophage), which is normally used in the management of diabetes mellitus, has also been shown to have a beneficial effect in: (check one)
Osteoporosis
Hyperthyroidism
Polycystic ovary syndrome
Right ventricular hypertrophy
Morbid truncal obesity

A

Polycystic ovary syndrome

Recent data suggest that insulin resistance and hyperinsulinemia are important in the pathogenesis of polycystic ovary syndrome (POS). Treatment with drugs that reduce insulin levels, such as metformin, has been shown to correct many of the metabolic abnormalities associated with POS. Such correction results in resumption of ovulation, decreased insulin resistance, and improved beta-cell function; it also produces improvement in cardiovascular risk factors such as dyslipidemia and impaired fibrinolysis.

37
Q

Which one of the following agents used for tocolysis has the unique adverse effect of respiratory depression? (check one)
Magnesium sulfate
Ritodrine (Yutopar)
Terbutaline (Brethine, Bricanyl)
Indomethacin (Indocin)
Nifedipine (Adalat, Procardia)

A

Magnesium sulfate

Magnesium sulfate infusions must be carefully monitored because respiratory depression is a potential lethal side effect. Reflexes are usually lost first. Terbutaline and ritodrine have the potential to cause respiratory distress in the form of pulmonary edema. They do not cause respiratory depression. Indomethacin and nifedipine are rarely used tocolytics that do not depress respiration.

38
Q

Compared to anesthesia using only parenteral opioids, the use of epidural anesthesia in labor and delivery increases the rate of which one of the following? (check one)
Cesarean section
Low Apgar scores (<7)
Maternal low backache 3 months post delivery
Prolonged second stage of labor

A

Prolonged second stage of labor

Multiple systematic reviews have been conducted to examine the effects of epidural anesthesia on maternal and neonatal outcomes. There are many confounding variables in the studies and, as a result, only a few effects of epidural anesthesia are consistently seen on a statistically significant basis: an increased duration of the second stage of labor, an increased rate of instrument-assisted vaginal deliveries, and an increased likelihood of maternal fever. Overall, there is no statistically significant difference in the duration of the first stage of labor, the incidence of low Apgar scores, or the incidence of maternal backache at 3 months or 12 months.

39
Q

An 18-year-old primigravida at 38 weeks’ gestation complains of a headache. Her blood pressure is 130/92 mm Hg. The fetal heart rate is 140 beats/min. A urine dipstick shows 2+ protein. Laboratory Findings Hemoglobin 10.8 g/dL (N 12.0–16.0) Hematocrit 32.4% (N 36.0–46.0) Platelets 110,000/mm3 (N 150,000–400,000) WBCs 14,900/mm3 (N 4000–10,000) Creatinine 0.5 mg/dL (N 0.8–1.3) AST (SGOT) 31 U/L (N 0–37) ALT (SGPT) 60 U/L (N 0–65) LDH 240 U/L (N 100–190) Bilirubin 1.9 mg/dL (N 0.0–1.0) A non stress test is reactive and the amniotic fluid index is 9.4 (N 8.0-20.0). The patient is admitted for further testing. After 24 hours repeat testing shows the following: Hemoglobin 9.8 g/dL Hematocrit 30.2% Platelets 92,000/mm3 WBCs 15,200/mm3 Creatinine 0.6 mg/dL AST (SGOT) 72 U/L ALT (SGPT) 98 U/L LDH 620 U/L Bilirubin 2.4 mg/dL 24-hour urine protein 2400 mg Which one of the following would be the most appropriate course of action at this point? (check one)
Continued monitoring, repeating the 24-hour urine collection, and repeating the laboratory studies tomorrow
Immediate delivery by cesarean section
Discharge to home on bed rest, with close follow-up
Induction of labor with oxytocin (Pitocin) if the cervix is favorable

A

Induction of labor with oxytocin (Pitocin) if the cervix is favorable

This patient has hemolysis, elevated liver enzymes, and low platelets (HELLP) syndrome and needs to be delivered. There is no reason to delay delivery in a term pregnancy. HELLP syndrome is a form of severe preeclampsia. If the patient has a favorable cervical examination, labor induction with oxytocin is appropriate. If the cervix is unfavorable, cesarean delivery should be considered to expedite delivery.

40
Q

A 30-year-old gravida 3 para 2 at 28 weeks’ gestation is a restrained passenger in a high-speed motor vehicle accident. After initial stabilization in the field with supplemental oxygen and intravenous fluids, she is brought into the emergency department on a backboard and wearing a cervical collar. Until you are able to rule out a spinal injury, in what position should the patient be kept? (check one)
Supine
Supine, with the uterus manually deflected laterally
Prone
Trendelenburg’s position
Left lateral decubitus

A

Supine, with the uterus manually deflected laterally

In general, it is best to place a woman who is greater than 20 weeks pregnant in the left lateral decubitus position because the uterus can compress the great vessels, resulting in decreased systolic blood pressure and uterine blood flow. However, in the case of trauma where a spinal cord injury cannot be ruled out, the woman needs to be kept supine on a backboard. The weight of the uterus can be shifted off the great vessels by either manual deflection laterally or by elevating the right hip 4–6 inches by placing towels under the backboard. The Trendelenburg position does not relieve the weight of the uterus on the great vessels. The prone position does not provide adequate spinal cord protection, and would be extremely awkward in a large pregnant woman.

41
Q

Which one of the following is true concerning nausea and vomiting in pregnancy? (check one)
Psychological factors play a causative role
Pharmacologic therapy, in general, is no more effective than placebo in relieving symptoms and preventing hospitalization
Metoclopramide (Reglan) is contraindicated in the first trimester but is safe and effective for nausea and vomiting in later trimesters
If nausea and vomiting begin after 9 weeks’ gestation, secondary causes are more likely to be present

A

If nausea and vomiting begin after 9 weeks’ gestation, secondary causes are more likely to be present

While the exact etiology of nausea and vomiting in pregnancy remains unclear, there are few data to support the theory that psychological factors play a role. Although nausea is usually a self-limited condition, other causes must be ruled out. Secondary causes are more likely to be present if the onset of symptoms occurs after 9 weeks’ gestation. Several pharmacologic treatments are proven safe and are superior to placebo in relieving symptoms and preventing hospitalization. Metoclopramide is more effective than placebo and has not been associated with an increased risk of adverse effects on the fetus.

42
Q

A 35-year-old female with previously regular menses presents with a 3-month history of amenorrhea, hot flashes, and increased irritability. A pregnancy test is negative, an estrogen level is low, and an FSH level is markedly elevated. There is no change in repeat testing 1 month later and you make a diagnosis of primary ovarian insufficiency. Further testing does not reveal a cause for her condition. She does not desire more children.

Which one of the following should you recommend to this patient for hormone replacement therapy? (check one)
No treatment
Transdermal estradiol without progestogen
Continuous oral estradiol without progestogen
Continuous oral estradiol and cyclic progestogen
Continuous oral estradiol and a levonorgestrel IUD (Mirena)

A

Continuous oral estradiol and a levonorgestrel IUD (Mirena)

Patients with primary ovarian insufficiency should be started on hormone replacement therapy (HRT) as soon as possible after diagnosis, preferably within a year. HRT treats symptoms and benefits bone health, cardiovascular health, and cognitive function. It should be continued until the age of expected natural menopause. In a patient with a uterus, protection against endometrial cancer with progesterone is required. A significant proportion of patients with this disorder occasionally ovulate or have spontaneous return of menses. HRT does not provide contraception. A levonorgestrel IUD provides both endometrial protection and contraception.

43
Q

A 28-year-old gravida 2 para 2 sees you for follow-up of heavy menstrual bleeding. She reports regular 30-day cycles with heavy bleeding that lasts for 5 days each month. She has tried ibuprofen and naproxen with minimal improvement. She and her husband desire more children but would prefer to wait a few years before trying again to conceive.

Which one of the following treatments is likely to be most effective in reducing her menstrual bleeding, while preserving future fertility? (check one)
An estrogen-progestin oral contraceptive
Tranexamic acid (Lysteda)
A levonorgestrel IUD (Mirena)
Uterine artery embolization
Endometrial ablation

A

A levonorgestrel IUD (Mirena)

Of the available treatment options, the levonorgestrel IUD is the most effective for reducing heavy menstrual bleeding (SOR A), with a 71%–95% decrease in menstrual blood loss expected. Estrogen-progestin oral contraceptives are less effective, with expected reduction in blood loss of 35%–69%, although they provide an added benefit of regulating menstrual bleeding in patients with anovulatory cycles. Nonhormonal medication options include tranexamic acid and NSAIDs, with an expected blood loss reduction of 26%–54% and 10%–52%, respectively. Two effective procedural options that are lower risk and less invasive compared to hysterectomy include uterine artery embolization and endometrial ablation, although both would present significant risk of reducing future fertility and therefore would not be the preferred treatment for this patient.

44
Q

A 31-year-old nulligravida presents to your office with an inability to conceive for the past 12 months. She reports irregular menses for the past 2 years. Her medical history is significant for Hashimoto thyroiditis that is currently controlled. A urine pregnancy test is negative. On examination you note vaginal dryness and labial atrophy. You suspect primary ovarian insufficiency. | Which one of the following combinations of FSH and LH levels is consistent with this diagnosis? (check one)
Normal FSH and normal LH
Low FSH and low LH
Low FSH and elevated LH
Elevated FSH and elevated LH

A

Elevated FSH and elevated LH

In a female younger than 40 years of age, elevated FSH and LH levels indicate primary ovarian insufficiency. Ovarian insufficiency leads to low estrogen levels, which stimulate increased production of FSH in the pituitary in a feedback loop. In this scenario, LH levels are high but do not rise as much as FSH levels. | Normal FSH and LH levels may indicate an outflow tract obstruction. Low FSH and LH levels indicate that the hypothalamic-pituitary axis is suppressed, as in the female athlete triad when there is an excess of energy expenditure compared to intake. Low FSH and elevated LH levels may be detected immediately prior to ovulation as part of a normal cyclical pattern.

45
Q

A new first-time mother calls for advice on nipple pain with breastfeeding. She is 6 days post partum after an uncomplicated delivery.
Which one of the following would be most effective?
(check one)
Lanolin cream
Expressed breast milk
Tea bag compresses
Hydrogel dressing
Education on positioning

A

Education on positioning

Nipple pain with breastfeeding is extremely common, with some studies reporting a prevalence of up to 96%. Preventing or alleviating nipple pain is important for comfort, but also for promoting breastfeeding in general. The best intervention for nipple pain is education on proper positioning and attachment of the infant. Topical remedies may also be effective, although no one topical agent has been shown to be clearly superior, and none is as effective as education on positioning and latch-on.

46
Q

You are evaluating a 25-year-old gravida 5 para 1 at 6 weeks estimated gestation. She has a history of three consecutive spontaneous miscarriages. Her workup has been negative except for a positive lupus anticoagulant on two occasions, separated by 6 weeks. You make the diagnosis of antiphospholipid antibody syndrome. She has no previous history of venous or arterial thrombosis. The best medical management at this time is: (check one)
Acetaminophen
Warfarin (Coumadin)
Prednisone
Aspirin and heparin combined
Progesterone

A

Aspirin and heparin combined

Antiphospholipid antibody syndrome in pregnancy is associated with an increased risk of thromboembolism, fetal loss, thrombocytopenia, and poor pregnancy outcome. Studies comparing aspirin alone versus aspirin and heparin suggest that the combination of aspirin and heparin is most effective for decreasing fetal loss. One study showed decreased fetal loss with a combination of corticosteroids and aspirin, but the results have not been reproduced in subsequent studies. In addition, the use of prednisone was associated with an increased risk of premature rupture of membranes, preterm delivery, fetal growth restriction, infection, preeclampsia, diabetes, osteopenia, and avascular necrosis. Progesterone may be useful for recurrent spontaneous abortion related to a luteal phase defect, but has not been shown to be effective in preventing complications associated with antiphospholipid antibody syndrome.

47
Q

A 26-year-old gravida 2 para 1 presents at 30 weeks gestation with a complaint of severe itching. She has excoriations from scratching in various areas. She says that she had the same problem during her last pregnancy, and her medical records reveal a diagnosis of intrahepatic cholestasis of pregnancy. Elevation of which one of the following is most characteristic of this disorder? (check one)
γ-Glutamyltransferase (GGT)
Bile acids
Direct bilirubin
Indirect bilirubin
Prothrombin time

A

Bile acids

Intrahepatic cholestasis of pregnancy classically presents as severe pruritus in the third trimester. Characteristic findings include the absence of primary skin lesions and elevation of serum levels of total bile acids. Jaundice and elevated bilirubin levels may or may not be present. The GGT usually is normal or modestly elevated, which can help differentiate this condition from other cholestatic liver diseases. The prothrombin time usually is normal, but if elevated it may reflect a vitamin K deficiency from malabsorption.

48
Q

A 36-year-old female with recently diagnosed polycystic ovary syndrome (PCOS) sees you to discuss treatment options. Her menses are irregular and she only has 2–3 cycles per year. She does not wish to have any more children. She has no contraindications to hormonal contraceptives. Her BMI is 27 kg/m2.

Which one of the following would be the most appropriate medication to initiate as first-line PCOS therapy? (check one)
Finasteride (Proscar)
Letrozole (Femara)
Metformin
Norgestimate/ethinyl estradiol (Sprintec)
Spironolactone (Aldactone)

A

Norgestimate/ethinyl estradiol (Sprintec)

While the second-line medications for polycystic ovary syndrome (PCOS) continue to shift as the evidence is refined, the primary goal of protecting the endometrial lining from unopposed estrogen effects remains consistent. The first-line therapy for PCOS in women who do not desire pregnancy remains combined oral contraception and/or progestin-containing contraceptives. Finasteride, letrozole, metformin, and spironolactone are second-line treatments for specific conditions caused by PCOS such as type 2 diabetes, anovulatory oligomenorrhea, infertility, and hirsutism.

49
Q

A 53-year-old female sees you because she would like treatment for hot flashes that she finds quite bothersome. Her last menstrual period was 8 months ago. She has a history of unprovoked deep vein thrombosis and a history of depression that is treated with venlafaxine (Effexor XR).

In addition to optimizing the dosage of her venlafaxine, which one of the following would be most effective for treatment of her hot flashes? (check one)
Black cohosh
Clonidine
Gabapentin (Neurontin)
Oral progesterone
Topiramate (Topamax)

A

Gabapentin (Neurontin)

Vasomotor symptoms such as hot flashes are experienced by 50%–75% of women during the menopausal transition. These are most effectively treated by systemic estrogen replacement therapy. For patients with an intact uterus, progesterone needs to be added for endometrial protection. In patients with contraindications to estrogen therapy, SSRIs and SNRIs have been shown to be 50%–65% effective and are often the next choice. Their mechanism of action remains unknown. Gabapentin has also been shown to reduce vasomotor symptoms by 40%–65%. Black cohosh and oral progesterone are not effective. Clonidine has some effect with vasomotor symptom reduction of 20%–40%. Topiramate has not been known to be effective.

50
Q

A 48-year-old female presents to your office because of painful sexual intercourse occurring with both initial entry and deep penetration. She has been using lubricants, but still experiences burning and dryness. Her last menstrual period was 1 year ago. A pelvic examination reveals slightly pale vaginal mucosa.

Which one of the following would be the best and most cost-effective initial therapy? (check one)
Vaginal estradiol cream (Estrace Vaginal)
Vaginal prasterone (Intrarosa)
Ospemifene (Osphena)
OnabotulinumtoxinA (Botox)
Pelvic floor physical therapy

A

Vaginal estradiol cream (Estrace Vaginal)

Dyspareunia is common in menopausal women and may be caused by vaginal atrophy that results in burning and dryness. Pain is generally caused by entry and deep penetration. An examination may show pale and dry vaginal mucosa, although this may appear normal in early menopause. It may be treated with vaginal moisturizers or lubricants, but further treatment may be needed. Topical estrogen is the most effective initial therapy and is much more cost-effective than other alternatives. For patients with a preference for non-estrogen treatment, ospemifene is a selective estrogen receptor modulator. However, it costs more than $200 per month and should not be used if there is a history of thromboembolism. A vaginal dehydroepiandrosterone formulation called prasterone decreases pain by increasing epithelial thickness, but also costs over $200 monthly. OnabotulinumtoxinA and pelvic floor physical therapy are utilized when pelvic floor dysfunction is the cause of dyspareunia.

51
Q

Which one of the following sonographic measurements is most accurate for estimating gestational age? (check one)
Amniotic sac size at 5 weeks of pregnancy
Crown-rump length at 10 weeks of pregnancy
Femur length at 16 weeks of pregnancy
Biparietal diameter at 20 weeks of pregnancy
Abdominal circumference at 24 weeks of pregnancy

A

Crown-rump length at 10 weeks of pregnancy

Estimation of gestational age by ultrasound is most accurate early in the first trimester and begins to decline by 22 weeks gestation. Crown-rump length is typically used to estimate gestational age before 13 weeks gestation. After 11 weeks gestation, combinations of biparietal diameter, femur length, head circumference, and abdominal circumference are used to estimate the gestational age. These factors are used by the software that generates ultrasonography reports.

52
Q

A 33-year-old gravida 2 para 2 presents with a 1-year history of amenorrhea, hot flashes, and vaginal dryness. She previously had normal menses and takes no medications. Her past medical and surgical histories are negative. The patient is 178 cm (70 in) tall and her BMI is 22 kg/m2. Her vital signs are normal. A physical examination is normal except for vaginal dryness. Laboratory studies reveal a negative urine pregnancy test, normal TSH and prolactin levels, and elevated LH and FSH levels.

The most likely diagnosis is (check one)
intrauterine synechiae (Asherman syndrome)
functional hypothalamic amenorrhea
polycystic ovary syndrome
primary ovarian insufficiency
Turner’s syndrome

A

primary ovarian insufficiency

This patient presents with secondary amenorrhea. The differential diagnosis includes polycystic ovary syndrome (PCOS), intrauterine synechiae (Asherman syndrome), functional hypothalamic amenorrhea, hypothyroidism, hyperprolactinemia, and primary ovarian insufficiency (also known as premature ovarian failure). This patient’s presentation and the laboratory findings are most consistent with a diagnosis of primary ovarian insufficiency. This is defined as menopause before the age of 40 due to ovarian follicular depletion. Laboratory findings will usually reveal a low serum estradiol and elevated FSH and LH levels. This condition is different than menopause because of the age of presentation and the unpredictability of long-term ovarian function (up to 10% of cases spontaneously remit and patients have a temporary return of fertility).

Patients with PCOS typically present with obesity, difficulty conceiving, and normal or low FSH and LH levels. This patient’s normal weight and prior history of normal menses make this diagnosis less likely. Intrauterine synechiae is characterized by scar tissue inside the uterus. Risk factors include intrauterine procedures, pregnancy, inflammation, and infection. Patients present with abnormal uterine bleeding, recurrent pregnancy loss, dysmenorrhea, and infertility. FSH and LH levels are usually normal.

Functional hypothalamic amenorrhea is characterized by suppression of the hypothalamic-pituitary-ovarian axis, usually due to extreme stress, excessive exercise, marked weight loss, and/or dysfunctional eating. LH and FSH levels are usually low or low-normal. Turner’s syndrome is caused by the 45,X genotype, and patients have short stature, a webbed neck, a low hairline, and cardiac abnormalities. This is unlikely in a patient who is 178 cm (70 in) tall and has a normal examination.

53
Q

A 30-year-old gravida 3 para 2 sees you for prenatal care at 13 weeks gestation. During her previous pregnancies she became hypertensive and had bilateral leg edema and proteinuria. These conditions resolved after delivery. Her only current medication is a prenatal vitamin.

In order to prevent this condition, which one of the following should be started today? (check one)
No new medications
Aspirin
Fish Oil
Magnesium
Vitamin C

A

Aspirin

Preeclampsia is diagnosed when the blood pressure is ≥ 140/90 mm Hg on two separate occasions after 20 weeks gestation, accompanied by proteinuria (>300 mg protein in a 24-hour urine collection or 2+ protein on a dipstick). If there is no protein in the urine, new-onset hypertension and the presence of any of the following would meet the criteria for preeclampsia: thrombocytopenia, renal insufficiency, impaired liver function, pulmonary edema, or cerebral or visual symptoms. This patient had preeclampsia during her previous pregnancies, which puts her at high risk for preeclampsia during her current pregnancy. Aspirin, 81 mg daily, is recommended for high-risk pregnant patients to prevent preeclampsia. Prophylaxis should begin after 12 weeks gestation and continue until delivery. Fish oil, magnesium, and vitamin C are not beneficial in the prevention of preeclampsia.

54
Q

A 20-year-old pregnant female presents to your clinic to discuss the possibility of delivering her baby at her home. A few of her friends have had their babies at home and she asks for your advice about her delivery.

Which one of the following presents the greatest risk with an out-of-hospital birth? (check one)
Nulliparity
Maternal age <25
Prior vaginal delivery
Single gestation
Cephalic presentation

A

Nulliparity

Of the options listed, any primigravida should be encouraged to consider a hospital delivery rather than an out-of-hospital delivery. Pregnant patients with a previous cesarean delivery, multiple gestation (twins or higher), or fetal malpresentation (breech or other) should also be strongly encouraged to deliver in a hospital setting (SOR B). Similarly, pregnant patients at 41 weeks gestation should be encouraged to pursue a hospital delivery (SOR B). The age of the mother is not relevant in this decision unless there are other related increased risks and comorbidities that need to be considered.

Patients who do plan a community birth should ensure that their maternity and neonatal health professionals are licensed and meet international Confederation of Midwives Global Standards for Midwifery Education, are practicing within an integrated and regulated health system, and have access to timely and safe transport to a hospital if necessary (SOR C). Unassisted childbirth should also be strongly discouraged (SOR C).

55
Q

A 35-year-old female presents with fatigue. She has been falling asleep at work for the past 6 weeks. She is married with two children and works as a nurse at the community hospital. Since she returned to work 12 weeks ago after maternity leave, her infant has had multiple respiratory infections and has not slept well through the night. Her menstrual cycle has been irregular and heavy for the past several months. A CBC and TSH level are normal.

Which one of the following laboratory tests would be appropriate at this visit? (check one)
25-Hydroxyvitamin D
β-hCG
D-dimer
A serum antibody test for Lyme disease

A

β-hCG

25-Hydroxyvitamin D levels should not be measured in patients presenting with fatigue (SOR A). A serum antibody test for Lyme disease or a D-dimer would not be indicated for this patient based on her history and symptoms. Because of the patient’s history of irregular menses, a β-hCG level would be indicated.

56
Q

A 27-year-old female with a past medical history of polycystic ovary syndrome (PCOS) would like to become pregnant. Which one of the following treatments for PCOS is associated with greater live-birth and ovulation rates? (check one)
Finasteride (Proscar)
Letrozole (Femara)
Metformin (Glucophage)
Spironolactone (Aldactone)

A

Letrozole (Femara)

In a double-blind randomized trial, letrozole was associated with greater live-birth and ovulation rates compared to clomiphene (SOR A). A Cochrane review indicated that metformin does not increase fertility in patients diagnosed with polycystic ovary syndrome (PCOS). Spironolactone and finasteride are both used to treat PCOS in women who do not desire pregnancy.

57
Q

Which one of the following is the most common cause of hirsutism in premenopausal women? (check one)
Idiopathic hirsutism
Polycystic ovary syndrome
Androgen-secreting tumors
Nonclassic congenital adrenal hyperplasia
Medication side effects

A

Polycystic ovary syndrome

The most common cause of hirsutism in premenopausal women is polycystic ovary syndrome, which
accounts for 75%–80% of cases. The second most common cause is idiopathic hirsutism, which accounts
for 5%–20% of cases. Other causes are rare, but should be considered when evaluating new cases of
hirsutism.

58
Q

A 17-year-old female comes to your office with an 8-month history of amenorrhea. Menarche occurred at age 12 and her menses were regular until the past year. On examination the patient’s vital signs are in the normal range for her age but she has a BMI of 16 kg/m2, which is below the third percentile for her age. She is a high school senior who dances with the local ballet company. She practices dance several hours a day and works out regularly. She tells you that she follows a strict 800-calorie/day diet to keep in shape for ballet.

You order a CBC, a comprehensive metabolic panel, a urine β-hCG level, FSH and LH levels, and a TSH level. Which one of the following is also recommended as part of the workup? (check one)
An EKG
Pelvic ultrasonography
Abdominal/pelvic CT
A DXA scan
A nuclear bone scan

A

A DXA scan

Relative energy deficiency in sport (RED-S), formerly known as the female athlete triad, is a relatively common condition in female athletes, and is characterized by amenorrhea, disordered eating, and osteoporosis. It is more common in sports that promote lean body mass. Female athletes should be screened for the disorder during their preparticipation evaluations. Individuals who present with one or more components of RED-S should be evaluated for the other components. This patient has a low BMI for her age, which indicates an eating disorder, and secondary amenorrhea, and should be screened for osteoporosis using a DXA scan. The International Society for Clinical Densitometry recommends using the Z-score, rather than the T-score, when screening children or premenopausal women. The T-score is based on a comparison to a young adult at peak bone density, whereas the Z-score uses a comparison to persons of the same age as the patient. A Z-score less than –2.0 indicates osteoporosis. The American College of Sports Medicine defines low bone density as a Z score of –1.0 to –2.0.

An EKG is not required in this patient since she has normal vital signs. Pelvic ultrasonography is not necessary unless an abnormal finding is identified on a pelvic examination. Abdominopelvic CT would be inappropriate given the patient’s age and lack of abdominopelvic symptoms such as pain or a mass. A nuclear bone scan likewise is not recommended, as it is not used to diagnose osteoporosis (SOR C).

59
Q

A 29-year-old gravida 2 para 1 comes to the hospital for scheduled induction of labor. Her last delivery was a spontaneous vaginal delivery without complications. Her pregnancy has been uneventful. Oxytocin (Pitocin) is used during induction according to the hospital protocol and her labor progresses without difficulty.

Which one of the following should be AVOIDED to minimize the risk of postpartum hemorrhage in this patient? (check one)
Administration of oxytocin with delivery of the anterior shoulder
Controlled cord traction
Active management of the third stage of labor
Routine episiotomy
Manual removal of a retained placenta

A

Routine episiotomy

Postpartum hemorrhage (PPH) is the cause of one-fourth of maternal deaths worldwide and 12% in the United States. It is defined as the loss of 1000 mL of blood or the loss of blood with coinciding signs and symptoms of hypovolemia within 24 hours after delivery. Twenty percent of PPH occurs in patients without risk factors, so methods to prevent this common problem should be in place with every delivery.
Active management of the third stage of labor (AMTSL) is crucial in the prevention of PPH. Administering oxytocin with or soon after the delivery of the anterior shoulder is the most important step of this process (SOR A). Even if oxytocin is used for induction, or as a part of AMTSL, it is still the most effective treatment for PPH (SOR A). Controlled cord traction is part of AMTSL and is necessary for the delivery of the placenta. If a retained placenta occurs it may be necessary to manually remove the placenta with necessary anesthesia. Trauma such as lacerations and episiotomies increases the risk of postpartum hemorrhage, so routine episiotomy should be avoided (SOR A).

60
Q

A healthy 49-year-old female presents to your office for a routine health maintenance visit. Since her last visit a year ago she has had only two menstrual periods. She reports sudden sensations of extreme heat in her face, neck, and chest that last just a few minutes but occur throughout the day. These symptoms are very bothersome and interfere with the quality of her sleep.

Which one of the following would you recommend to relieve her symptoms? (check one)
Black cohosh
Combined estrogen and progesterone
Compounded bioidentical hormones
Micronized progesterone

A

Combined estrogen and progesterone

This patient presents with typical vasomotor symptoms that can begin in perimenopause and affect sleep
quality. Hormone therapy is the gold standard for treatment of vasomotor symptoms. Combination estrogen
and progesterone therapy is highly effective for vasomotor symptoms and provides protection against
uterine neoplasia. Although micronized progesterone decreases vasomotor symptoms there are no
long-term studies to assess the safety of progestin-only treatment for menopausal symptoms.
Compounded bioidentical hormone therapy creates safety concerns and is not a first-line therapy due to
limited government regulation and monitoring, the potential for overdosing and underdosing, impurities
or lack of sterility, and the lack of labeling describing risks. Testosterone alone is not FDA-approved for
use in women. Additionally, it has not been shown to be beneficial for treatment of vasomotor symptoms
in combination with hormone therapy and is associated with significant side effects. It may be useful for
hypoactive sexual desire in postmenopausal women. There is insufficient data to recommend the use of
herbal remedies such as black cohosh.

61
Q

A 38-year-old patient wishes to start contraception. She currently takes lisinopril (Prinivil, Zestril) for hypertension and also takes sumatriptan (Imitrex) occasionally for migraines at the first sign of flashing lights or zigzagging lines in her vision. Her medical, family, and social histories are otherwise unremarkable. An examination is notable only for a blood pressure of 130/80 mm Hg and a BMI of 36 kg/m2.

The patient is interested in using either the vaginal ring or the contraceptive patch. Which one of the following would you recommend? (check one)
Transdermal norelgestromin/ethinyl estradiol (Ortho Evra)
The etonogestrel/ethinyl estradiol vaginal ring (NuvaRing)
Neither method due to her migraines
Neither method due to her age
Losing weight before starting either method

A

Neither method due to her migraines

Family physicians are often asked to provide contraception and need to be familiar with the current methods and contraindications. Estrogen-containing products, including the contraceptive patch and the vaginal ring, are contraindicated in smokers >35 years of age and in patients with migraine with aura.

62
Q

An asymptomatic 42-year-old female sees you for a routine evaluation. On examination her uterus is irregularly enlarged to the size seen at approximately 8 weeks gestation. Pelvic ultrasonography shows several uterine fibroid tumors measuring <5 cm. The patient does not desire future fertility.

Which one of the following would be the most appropriate management option? (check one)
Observation only
An oral contraceptive
A gonadotropin-releasing hormone (GnRH) agonist
Laparoscopic myomectomy
Hysterectomy

A

Observation only

Uterine fibroid tumors (leiomyomas) are the most common tumors of the female reproductive tract, with some evidence suggesting that the cumulative incidence in women age 25–45 years is approximately 30%. Symptoms related to fibroids can include menorrhagia, pelvic pain, obstructive symptoms, infertility, or pregnancy loss. However, many fibroids are asymptomatic and are discovered incidentally, with observation being the preferred management in this situation (SOR B). The risk of malignant leiomyosarcoma is exceedingly small (0.23% in one study) and there is a risk of side effects or complications from other treatment modalities.

For women who are symptomatic, the data is insufficient regarding the most appropriate therapy. Surgical options include myomectomy, hysterectomy, uterine artery embolization, and myolysis, but data to allow direct comparison is lacking. With the exception of trials of GnRH-agonist therapy as an adjunct to surgery, there is not enough randomized trial data to support the use of medical therapies such as oral contraceptives, NSAIDs, or progestins in the treatment of symptomatic fibroids.

63
Q

A 25-year-old female sees you because of irregular menses, hirsutism, and moderate acne. She is sexually active in a monogamous relationship with a male, has never been pregnant, and prefers not to become pregnant at this time.

Which one of the following is considered first-line therapy?

(check one)
Clomiphene (Clomid)
Letrozole (Femara)
Levonorgestrel/ethinyl estradiol
Metformin (Glucophage)
Spironolactone (Aldactone)

A

Levonorgestrel/ethinyl estradiol

The Endocrine Society recommends hormonal contraception as the first-line medication for women diagnosed with polycystic ovary syndrome (PCOS) who are experiencing irregular menses, acne, and hirsutism and do not desire pregnancy (SOR A). Metformin may help regulate menses but has not been shown to be as effective as oral hormone therapy. In a 2015 Cochrane review, oral contraceptives were recommended as the most effective treatment for hirsutism. Either letrozole or clomiphene is appropriate for women diagnosed with PCOS who want to become pregnant.

64
Q

A 63-year-old female presents with a 3-week history of vulvar itching and burning. The patient has not had any vaginal bleeding since her last period at age 51.

An examination reveals thin, dry labia minora and sparse pubic hair. The vaginal mucosa is smooth and pale with a frothy yellow discharge. A point-of-care urinalysis shows moderate leukocytes and trace protein. Microscopy of vaginal secretions reveals motile flagellates.

Which one of the following medications is most likely to resolve this patient’s symptoms? (check one)
Estradiol vaginal cream, 2 g vaginally daily
Fluconazole (Diflucan), 150 mg as a single dose
Doxycycline, 100 mg twice daily for 7 days
Metronidazole, 500 mg twice daily for 7 days
Sulfamethoxazole/trimethoprim (Bactrim), 800/160 mg twice daily for 7 days

A

Metronidazole, 500 mg twice daily for 7 days

This patient presents with symptoms suggestive of vaginitis, which can be caused by vaginal infections, atrophy, and irritation from pads, panty liners, soaps, and perfumes. Atrophic vaginitis due to lack of estrogen is common in postmenopausal females. The condition responds to estrogen therapy, but infectious causes must first be ruled out. The presence of motile flagellates confirms the diagnosis of Trichomonas vaginitis, which would be treated with oral metronidazole, either as a one-time 2-g dose or 500 mg twice daily for 7 days. Estradiol, fluconazole, doxycycline, and sulfamethoxazole/trimethoprim are not used for the treatment of Trichomonas.

65
Q

A 30-year-old female with anovulatory uterine bleeding asks about treatment options. An examination is normal and blood testing is negative. She is unmarried and is undecided about having children.

Which one of the following would be the most appropriate treatment for this patient? (check one)
Oral progestin during the luteal phase
A levonorgestrel-releasing IUD
Endometrial ablation
Hysterectomy

A

A levonorgestrel-releasing IUD

IFew treatments for dysfunctional uterine bleeding have been studied. NSAIDs, oral contraceptive pills, and danazol have not been shown to have sufficient evidence of effect for the treatment of dysfunctional uterine bleeding. Progestin is effective when used on a 21-day cycle, but not if used only during the luteal phase. Hysterectomy and ablation are very effective, but both eliminate fertility. In a young woman unsure about having children, the levonorgestrel-releasing IUD is the most effective treatment that preserves fertility (SOR A).

66
Q

During a routine health maintenance visit a 38-year-old female expresses concern about her risk of breast cancer because her mother and another relative have had breast cancer. She is asymptomatic and your clinical breast examination reveals no masses.

For this patient, the U.S. Preventive Services Task Force recommends which one of the following? (check one)
Administering a familial risk stratification tool
BRCA mutation testing
Bilateral screening mammography
MRI of the breasts
Referral for genetic counseling

A

Administering a familial risk stratification tool

Mammographic screening is not recommended at the age of 38. The U.S. Preventive Services Task Force (USPSTF) recommends against routine mammographic screening for breast cancer between the ages of 40 and 49, but promotes a policy of individualized shared decision making. Mammography every 2 years is recommended for women between the ages of 50 and 74 (B recommendation). The USPSTF recommends any of several familial risk stratification tools for use in women who have a family member with breast, ovarian, tubal, or peritoneal cancer, to assess for an increased risk of a BRCA1 or BRCA2 mutation. If the screen is positive, a referral for genetic counseling is recommended to determine if BRCA testing is indicated (B recommendation). MRI of the breasts is not recommended for screening by the USPSTF but is recommended by some organizations as an adjunct to mammography for women determined to be at high risk of breast cancer, such as those with a BRCA mutation.

67
Q

A 37-year-old female who smokes 1 pack of cigarettes per day has just had her third child and requests contraception. She does not intend to have any more children.

Which one of the following is the safest option for this patient? (check one)
Traditional combined oral contraceptive pills
Extended-cycle combined oral contraceptive pills
The contraceptive patch (Ortho Evra)
The contraceptive vaginal ring (NuvaRing)
The etonogestrel implant (Nexplanon)

A

The etonogestrel implant (Nexplanon)

Cigarette smoking, increasing age, and exogenous estrogen, particularly at the supraphysiologic doses used in contraceptives, all increase risk for vascular events such as venous thromboembolism and stroke. The use of estrogen-containing contraception in smokers ≥35 years is contraindicated because of this risk. All of the contraceptive options listed contain estrogen except for the etonogestrel implant. Other progestin-only contraceptive options that could be considered include depot medroxyprogesterone acetate, the levonorgestrel-releasing IUD, and progestin-only oral contraceptive pills.

68
Q

A 47-year-old female is concerned about a change in her menstrual pattern. Her monthly periods continue, but for the past several months they have been heavier than usual and have been lasting a few days longer. Last month she also noted some spotting for several days prior to the onset of her menses. Her pelvic examination is normal.

Which one of the following would be most appropriate at this time? (check one)
Observation only, and reexamination in 3 months
A serum FSH level
Transvaginal ultrasonography
Progestin-only therapy to normalize bleeding
Cyclic estrogen-progestin therapy to normalize bleeding

A

Transvaginal ultrasonography

Abnormal uterine bleeding can be a sign of endometrial cancer in premenopausal women, who account for 20% of cases of endometrial cancer. The American College of Obstetricians and Gynecologists recommends that women with abnormal uterine bleeding should be evaluated for endometrial cancer if they are older than 45 years or if they have a history of unopposed estrogen exposure (SOR C). Most guidelines recommend either transvaginal ultrasonography or endometrial biopsy as the initial study in the evaluation of endometrial cancer. Transvaginal ultrasonography is often preferred as the initial study because of its availability, cost-effectiveness, and high sensitivity. If bleeding persists despite normal transvaginal ultrasonography a tissue biopsy should be performed. The listed hormonal treatment options may be appropriate once cancer is ruled out. An FSH level can help determine whether someone is menopausal or approaching menopause, in which case they will likely be missing periods. Continued observation would only delay the diagnosis.

69
Q

A 45-year-old female comes to your outpatient clinic for a routine health maintenance examination. She has no complaints, is generally healthy, and has an unremarkable family history. She has been having regular periods and is sexually active with her husband, who had a vasectomy 9 years ago.

As you perform a pelvic examination you notice a fullness in the left adnexal region that is mildly tender to palpation. The remainder of the examination is normal. You order pelvic ultrasonography, which demonstrates a 4×5-cm cystic structure on the left ovary. There are no internal septations or echogenic internal structures. A pregnancy test is negative.

Which one of the following would be most appropriate at this point? (check one)
Reassurance only
Repeat pelvic ultrasonography in 8–10 weeks
A serum CA-125 level
An oral contraceptive and repeat pelvic ultrasonography in 6–8 months
Referral to a gynecologist for diagnostic cystectomy

A

Repeat pelvic ultrasonography in 8–10 weeks

Functional ovarian cysts are estimated to be present in approximately 15% of menstruating women and are often found incidentally on a pelvic examination or on diagnostic imaging performed for other reasons. The majority of these lesions resolve spontaneously within two or three menstrual cycles and management should consist of follow-up ultrasonography 8–12 weeks after the cyst is identified. Cysts that are particularly symptomatic, have less than simple features, are >10 cm in size, or that persist longer than 12 weeks should be referred for consideration of diagnostic removal (cystectomy or oophorectomy depending on patient characteristics). Patients who carry a BRCA gene or who have a family history of ovarian cancer should also be considered for invasive testing sooner. Serum CA-125 testing is difficult to interpret in premenopausal patients, who frequently have elevated levels without evidence of a malignancy. In postmenopausal women with an ovarian cyst a CA-125 level >35 U/mL should prompt referral for removal.

70
Q

Which one of the following populations should be screened for asymptomatic bacteriuria? (check one)
Patients with diabetes mellitus
Patients who are pregnant
Hospitalized patients who have an indwelling Foley catheter
Hospitalized patients at the time of removal of an indwelling Foley catheter
Residents of long-term care facilities

A

Patients who are pregnant

The U.S. Preventive Services Task Force (USPSTF) recommends screening for asymptomatic bacteriuria with a urine culture for pregnant women at 12–16 weeks gestation or at the first prenatal visit if it occurs later (B recommendation). The USPSTF recommends against screening for asymptomatic bacteriuria in men and nonpregnant women (D recommendation).

The Infectious Diseases Society of America (IDSA) guidelines for asymptomatic bacteriuria recommend that pregnant women should be screened for bacteriuria by urine culture at least once in early pregnancy, and they should be treated if the results are positive (B-I recommendation). The IDSA also recommends screening for and treatment of asymptomatic bacteriuria before transurethral resection of the prostate (B-I recommendation) and also before other urologic procedures for which mucosal bleeding is anticipated (A-III recommendation).

The IDSA’s recommendation for the Choosing Wisely campaign is to not treat asymptomatic bacteriuria with antibiotics. The only exceptions to this recommendation include pregnant women, patients undergoing prostate surgery or other invasive urologic surgery, and kidney or kidney pancreas organ transplant patients within the first year of receiving the transplant.

Screening for asymptomatic bacteriuria is not recommended for long-term care residents or patients with indwelling bladder catheters without symptoms of a UTI (catheter-associated asymptomatic bacteriuria).

71
Q

A 20-year-old female visits your office for advice regarding contraceptives. She expresses interest in long-acting reversible contraception (LARC).

Which one of the following is the recommended timing for LARC placement? (check one)
Any time during the menstrual cycle
14 days before the anticipated onset of menses
7 days before the anticipated onset of menses
7 days after the onset of menses
14 days after the onset of menses

A

Any time during the menstrual cycle

Long-acting reversible contraception (LARC) includes the copper IUD, levonorgestrel IUDs, and subdermal implants. LARCs can be placed at any point in the patient’s menstrual cycle (SOR A). There should be evidence that the patient is not pregnant prior to placement.

72
Q

A 22-year-old female with polycystic ovary syndrome comes in to discuss contraception. She has no other health conditions and takes no medications. Her menses are somewhat irregular, occurring every 28–42 days. She also asks about treatment for her mild hirsutism.

Which one of the following medications would be most likely to address her need for contraception and also improve her hirsutism? (check one)
Spironolactone (Aldactone)
Cyclic progesterone
Progesterone-only contraceptive pills
Oral combined hormonal contraceptives
A levonorgestrel-releasing IUD (Mirena)

A

Oral combined hormonal contraceptives

Management of polycystic ovary syndrome is typically aimed at addressing patient symptoms, as well as irregular menses and the risk of endometrial hyperplasia. Infertility may become a therapeutic target for women who desire pregnancy at some point in their lives. In this patient, who needs contraception and hopes to address her hirsutism, combined oral contraceptives are most likely to address both concerns. In addition to suppressing ovulation they also suppress gonadotropin and ovarian androgen production. The estrogen component increases hepatic production of sex hormone binding globulin, thus decreasing androgen bioavailability.

Progestin-only pills and the levonorgestrel IUD protect against pregnancy but will not improve hirsutism. Cyclic progesterone every 1–3 months can be used to prevent endometrial hyperplasia but will not provide contraception or address hirsutism. Spironolactone is an androgen receptor antagonist that can decrease hair growth, but it will not provide contraception.

73
Q

A 20-year-old female presents to your office with questions about her contraceptive method. She has been using a combined oral contraceptive pill for the past 2 years without any complications. She has learned that several of her friends recently switched to an IUD. She is concerned about the efficacy of her current method and asks about the failure rate.

You tell her that with typical use, the annual failure rate of a combined oral contraceptive pill is (check one)
0.2%
2%
7%
18%
22%

A

7%

The annual failure rate of combined oral contraceptive pills with typical use is 7%. Typical failure rates for other contraceptive methods are 0.2% for the levonorgestrel IUD, 4% for injectable progestin, 13% for male condoms, and 22% for the withdrawal method.

74
Q

A 42-year-old premenopausal female presents to your office with new-onset bilateral nipple discharge for the past 4 weeks. She describes the discharge as green and nonbloody. She has a past medical history of diabetes mellitus, dyslipidemia, hypertension, and depression. Her current medications include the following:

Atorvastatin (Lipitor)
Escitalopram (Lexapro)
Hydrochlorothiazide
Lisinopril (Zestril)
Metformin

Her vital signs are unremarkable. A physical examination is significant for nonbloody green fluid expressed from the nipples. A TSH level, comprehensive metabolic panel, and CBC are all within normal range, and a serum hCG test is negative. A prolactin level is elevated at 85 ng/mL (N <30 in nonpregnant premenopausal females).

Which one of her medications is most likely to cause galactorrhea? (check one)
Atorvastatin
Escitalopram
Hydrochlorothiazide
Lisinopril
Metformin

A

Escitalopram

Of this patient’s medications, escitalopram is most likely to induce galactorrhea. SSRIs are responsible for 95% of medication-induced galactorrhea cases. The etiology of an elevated prolactin level <100 ng/mL is commonly medication, systemic pathology, or a microadenoma. Macroadenomas are associated with higher prolactin levels (>250 ng/mL). A normal physical examination, negative hCG level, and unremarkable TSH level, BUN level, creatinine level, and liver function tests further support a medication-induced etiology for this patient’s galactorrhea. Antihypertensives such as calcium channel blockers and methyldopa may cause galactorrhea, while diuretics such as hydrochlorothiazide and ACE inhibitors such as lisinopril are not known offenders. Neither atorvastatin nor metformin are common etiologies for medication-induced hyperprolactinemia, although atorvastatin can cause gynecomastia.

75
Q

A 34-year-old sexually active female consults you about contraception options. She has late-stage kidney disease and her nephrologist has notified you that she will likely be recommended for kidney transplantation soon.

Which one of the following would you recommend for safety and efficacy? (check one)
Condoms
Combined oral contraceptive pills
Medroxyprogesterone acetate (Depo-Provera) injections
An IUD
An etonogestrel/ethinyl estradiol vaginal ring (NuvaRing)

A

An IUD

Family physicians are often asked to provide primary care for organ transplant recipients. Pregnancy should be avoided during the 12 months following transplantation because of the increased risk of preterm delivery and graft rejection. Female fertility typically increases post transplant. The use of an IUD avoids interactions with medications, does not increase the risk of infection, and is not affected by typical immunosuppressive therapies.

The remaining options are incorrect because of their higher failure and discontinuation rates. The CDC cites failure rates with typical use of 9% for combined oral contraceptives and the etonogestrel/ethinyl estradiol vaginal ring, 6% for injectable progesterone, 0.2% for levonorgestrel IUDs, and 0.08% for the copper IUD. Barrier method failure rates exceed 18%. Combined oral contraceptives and the vaginal ring also have potential estrogen-related side effects, and injectable progesterone use increases the risk for osteoporosis.

76
Q

A 52-year-old female sees you because of a vaginal discharge. An examination reveals a malodorous, greenish-yellow, frothy discharge, and inflammation of the cervix and vagina.

Which one of the following is the most likely diagnosis? (check one)
Atrophic vaginitis
Irritant/allergic vaginitis
Bacterial vaginosis
Trichomoniasis
Vulvovaginal candidiasis

A

Trichomoniasis

Trichomoniasis classically presents as a greenish-yellow, frothy discharge with a foul odor. Erythema and
inflammation of the vagina and cervix are often present and can include punctate hemorrhages (strawberry
cervix). Atrophic vaginitis may cause a thin, clear discharge and is usually associated with a thin, friable
vaginal mucosa. Irritant/allergic vaginitis causes burning and soreness with vulvar erythema but usually
does not cause any significant discharge. Bacterial vaginosis more commonly presents as a thin,
homogenous discharge with a fishy odor and no cervical or vaginal inflammation. Vulvovaginal candidiasis
presents with white, thick, cheesy, or curdy discharge.

77
Q

A 15-year-old female presents for a well adolescent examination and reports painful, heavy periods. Menarche occurred at age 11, and by age 12 her menses were regular but quite painful. She misses at least 1 day of school each month due to the discomfort. She has tried acetaminophen/caffeine/pyrilamine (Midol Complete) and ibuprofen, 200 mg, without much relief. She is not sexually active. Her past medical history and surgical history are unremarkable. A urine pregnancy test is negative.

Which one of the following would be the most appropriate next step for this patient? (check one)
Empiric treatment with maximum-dose naproxen
Screening for sexually transmitted infections
A pelvic examination
Pelvic ultrasonography
Referral to a gynecologist

A

Empiric treatment with maximum-dose naproxen

Dysmenorrhea affects 50%–90% of females and the great majority of cases are primary dysmenorrhea, or pain that occurs in the absence of pelvic pathology. After a complete history confirming cyclic cramping pelvic pain beginning around the start of menses and a negative urine pregnancy test, empiric treatment should be offered (SOR C). First-line treatment is an NSAID at moderate to maximum dosing, such as naproxen, 500 mg every 12 hours. Any NSAID can be used and should be started 1–2 days before the onset of menses and continued through the first several days of bleeding. A secondary benefit to NSAID use is a reduction in heavy menstrual bleeding. Combined estrogen/progestin oral contraceptives may also be used as first-line therapy or in conjunction with NSAIDs.

While screening for sexually transmitted infections is important for sexually active adolescents, it is not indicated in the evaluation of dysmenorrhea. Neither pelvic examination nor imaging is indicated when the history is consistent with primary dysmenorrhea. If there is evidence of secondary dysmenorrhea (due to pelvic pathology or a recognized medical condition), then an examination and imaging are indicated. Family physicians are able to manage the majority of cases of primary dysmenorrhea. If there is no improvement in treatment after 3 months, referral to a gynecologist may be indicated.

78
Q

A 24-year-old gravida 2 para 2 presents to your office for evaluation of breast pain. The patient reports that the pain affects both breasts, particularly during the week prior to her menses. She does not report any redness, nipple discharge, or fever. This is the first time the patient has presented for evaluation of this problem.

In addition to conservative measures, which one of the following would be most appropriate to manage the patient’s pain without interfering with fertility? (check one)
Diclofenac gel (Voltaren Arthritis Pain)
Danazol
Goserelin (Zoladex)
Tamoxifen (Soltamox)

A

Diclofenac gel (Voltaren Arthritis Pain)

Mastalgia is a common symptom requiring evaluation in the primary care setting. Cyclic mastalgia accounts for about two-thirds of all breast pain and is thought to be caused by increased sensitivity of the breast tissues to hormonal stimulation during the luteal phase of the menstrual cycle. Topical NSAIDs such as diclofenac are the first-line pharmacologic treatment for mastalgia (SOR B). Danazol is the only drug that is approved by the FDA for treatment of mastalgia, but it is poorly tolerated due to menorrhagia, muscle cramps, weight gain, and other androgenic effects. Goserelin is only indicated for severe, refractory mastalgia. Tamoxifen is more effective and better tolerated than danazol, but is associated with hot flashes, vaginal discharge, venous thromboembolism, endometrial cancer, and teratogenicity.

79
Q

The majority of U.S. women who unintentionally become pregnant choose to (check one)
carry the pregnancy to term and keep the baby
carry the pregnancy to term and place the baby for adoption
terminate the pregnancy by medical means legally
terminate the pregnancy by surgical means legally
terminate the pregnancy illegally

A

carry the pregnancy to term and keep the baby

While recent data is not available, 45% of pregnancies in the United States in 2011 were unintentional.
About 42% of pregnant women with unintended pregnancies chose to terminate the pregnancy by one
means or another. Of the 58% that chose to carry the pregnancy to term, only about 1% placed the infant
for adoption.

80
Q

According to U.S. Preventive Services Task Force guidelines, which one of the following interventions is recommended after 12 weeks gestation in women who are at high risk for preeclampsia? (check one)
Aspirin, 81 mg daily
Magnesium gluconate, 1500 mg daily
Fish oil, 1000 mg daily
Vitamin C, 1000 mg daily
Vitamin D, 2000 IU daily

A

Aspirin, 81 mg daily

For women at high risk of developing preeclampsia, the U.S. Preventive Services Task Force (USPSTF)
recommends starting low-dose aspirin after 12 weeks gestation (B recommendation). While calcium
appears to be helpful in preventing preeclampsia for women with a diet deficient in calcium, the evidence
is not yet conclusive. The USPSTF recommendation does not address the use of fish oil, magnesium
gluconate, vitamin C, or vitamin D for the prevention of preeclampsia.

81
Q

A 38-year-old female presents for evaluation of infertility after being unable to conceive for the past 14 months. She has a history of type 2 diabetes, obesity, and hypothyroidism, and takes metformin (Glucophage) and levothyroxine (Synthroid) daily. A review of systems is notable for menses that occur once every 35–50 days and persistent dark hair growth on her chin and areolae. An examination is remarkable only for a blood pressure of 142/95 mm Hg, a BMI of 37 kg/m2, and the hair growth described by the patient.

Which one of the findings in this patient is a required diagnostic criterion for polycystic ovary syndrome? (check one)
Hyperandrogenism
Hypertension
Hypothyroidism
Infertility
Obesity

A

Hyperandrogenism

Several professional organizations have published criteria for the diagnosis of polycystic ovary syndrome
(PCOS) using various combinations of hyperandrogenism (clinical or biochemical), ovulatory dysfunction
(typically oligomenorrhea), and the presence of at least one polycystic ovary by imaging criteria.
Hypertension, hypothyroidism, infertility, and obesity are common symptoms in patients with PCOS but
are not diagnostic.
The National Institutes of Health advises that patients must have both hyperandrogenism and
oligomenorrhea to meet the criteria for PCOS. The Endocrine Society recommends diagnosing PCOS
based on the 2003 Rotterdam criteria, which require the presence of two of the following:
hyperandrogenism, ovulatory dysfunction, and at least one polycystic ovary. The Androgen Excess and
PCOS Society says that patients must have hyperandrogenism plus either oligomenorrhea or at least one
polycystic ovary for the diagnosis to be made.

82
Q

A 60-year-old female has recently moved to your area from another state and sees you to establish care. Her past records are not available at the time of her visit but she has a history of ER+/PR+ breast cancer diagnosed 2 years ago. She was treated with lumpectomy, radiation, and chemotherapy. She does not have any current symptoms and there is no family history of breast or ovarian cancer. Her last visit with a physician was 6 months ago. She had a mammogram at that time. A breast MRI was done at the time of her cancer diagnosis. Her last Papanicolaou (Pap) test was normal 1 year ago.

Which one of the following should be performed at this time? (check one)
A history and physical examination only
A Pap test
Mammography
Breast MRI
Echocardiography

A

A history and physical examination only

Breast cancer survivors should undergo a history and physical examination every 3–6 months for the first
3 years after treatment, then every 6–12 months for the next 2 years, and then annually thereafter (SOR
C).
Papanicolaou testing guidelines do not change for patients with a history of breast cancer. Screening should
be repeated every 3–5 years according to American Society for Colposcopy and Cervical Pathology
(ASCCP) guidelines.
Mammograms of both breasts or the remaining breast are recommended no more often than yearly (SOR
A). Breast MRI is not recommended on a regular basis unless the patient has a high risk of recurrence,
a significant family history of breast or ovarian cancer, or a personal history of Hodgkin’s disease (SOR
C). While it is important to be alert for signs of cardiotoxicity due to prior chemotherapy,
echocardiography is indicated only if the patient has cardiac symptoms. Routine echocardiography is not
recommended (SOR C).

83
Q

A 30-year-old gravida 3 para 2 sees you for prenatal care at 13 weeks gestation. During her previous pregnancies she became hypertensive and had bilateral leg edema and proteinuria. These conditions resolved after delivery. Her only current medication is a prenatal vitamin.

In order to prevent this condition, which one of the following should be started today? (check one)
No new medications
Aspirin
Fish oil
Magnesium
Vitamin C

A

Aspirin

Preeclampsia is diagnosed when the blood pressure is 140/90 mm Hg on two separate occasions after 20 weeks gestation, accompanied by proteinuria (>300 mg protein in a 24-hour urine collection or 2+ protein on a dipstick). If there is no protein in the urine, new-onset hypertension and the presence of any of the following would meet the criteria for preeclampsia: thrombocytopenia, renal insufficiency, impaired liver function, pulmonary edema, or cerebral or visual symptoms. This patient had preeclampsia during her previous pregnancies, which puts her at high risk for preeclampsia during her current pregnancy. Aspirin, 81 mg daily, is recommended for high-risk pregnant patients to prevent preeclampsia. Prophylaxis should begin after 12 weeks gestation and continue until delivery. Fish oil, magnesium, and vitamin C are not beneficial in the prevention of preeclampsia.

84
Q

A gravida 2 para 0 at 34 weeks gestation presents to your office because of diffuse itching. She does not have any known allergies other than seasonal allergies, and she does not have any new contacts. An examination is normal other than some scattered excoriations, and there is no other distinct rash. She has tried moisturizers but her symptoms have not improved.

Which one of the following would be most appropriate at this point? (check one)
Monitoring for the development of a rash
Liver function tests and serum bile acid levels
Topical corticosteroids
Oral antihistamines
Varicella-zoster immune globulin

A

Liver function tests and serum bile acid levels

Whenever a pregnant woman presents with pruritus without a primary rash, it is important to evaluate her
for intrahepatic cholestasis of pregnancy. This diagnosis is associated with increased fetal mortality and
warrants increased antenatal surveillance as well as possible induction by 35–37 weeks gestation. It is most
appropriate to check for elevation of liver function tests and serum bile acids. Emollients, topical
corticosteroids, and oral antihistamines can all be helpful for pruritus and certain rashes, but in this patient
it is most important to promptly look for the cause of the pruritus. Varicella-zoster immune globulin would
be indicated if she had no immunity to varicella and had been exposed to varicella or if she had a rash that
was suspected to be chickenpox.